MathGroup Archive 2005

[Date Index] [Thread Index] [Author Index]

Search the Archive

Re: Trouble with an integral

  • To: mathgroup at smc.vnet.net
  • Subject: [mg57518] Re: Trouble with an integral
  • From: "Valeri Astanoff" <astanoff at yahoo.fr>
  • Date: Tue, 31 May 2005 04:59:11 -0400 (EDT)
  • References: <d741q1$iff$1@smc.vnet.net><d76np2$7lh$1@smc.vnet.net>
  • Sender: owner-wri-mathgroup at wolfram.com

Seems that no close form is available.

I just got an equivalent infinite sum in terms of beta functions :

(4-Sqrt[2])/3 + Sum[(-1)^(n + 1)*a^n*Beta[-1, 2*n+1, 3/2]/(Sqrt[2]*n!),
 {n, 1, Infinity}]

Thanks anyway for your help.

v.a.


  • Prev by Date: Re: sum the elements of a list
  • Next by Date: Re: sum the elements of a list
  • Previous by thread: Re: Trouble with an integral
  • Next by thread: How to get decent quality plots.